How to prove that if $aequiv b pmod{kn}$ then $a^kequiv b^k pmod{k^2n}$












2














What I have done is this:



$aequiv b pmod{2n}$,



$a=b+ctimes2n$, for some $c$,



$a^2=b^2+2btimes ctimes2n+c^2times2^2n^2$,



$a^2-b^2=(btimes c+c^2n)times4n$, then



$a^2equiv b^2pmod{2^2n}$.



I think that this is right: what I DON’T understand is how to generalize this to:



$aequiv bpmod{kn}Rightarrow a^kequiv b^k pmod{k^2n}$.



Please give me a hint.










share|cite|improve this question















migrated from crypto.stackexchange.com Dec 1 '11 at 13:30


This question came from our site for software developers, mathematicians and others interested in cryptography.















  • Hi @gurghet - I've migrated your question here as it has no direct relation to cryptography as is - so this is the best place to get a good answer.
    – user892
    Dec 1 '11 at 13:32
















2














What I have done is this:



$aequiv b pmod{2n}$,



$a=b+ctimes2n$, for some $c$,



$a^2=b^2+2btimes ctimes2n+c^2times2^2n^2$,



$a^2-b^2=(btimes c+c^2n)times4n$, then



$a^2equiv b^2pmod{2^2n}$.



I think that this is right: what I DON’T understand is how to generalize this to:



$aequiv bpmod{kn}Rightarrow a^kequiv b^k pmod{k^2n}$.



Please give me a hint.










share|cite|improve this question















migrated from crypto.stackexchange.com Dec 1 '11 at 13:30


This question came from our site for software developers, mathematicians and others interested in cryptography.















  • Hi @gurghet - I've migrated your question here as it has no direct relation to cryptography as is - so this is the best place to get a good answer.
    – user892
    Dec 1 '11 at 13:32














2












2








2


0





What I have done is this:



$aequiv b pmod{2n}$,



$a=b+ctimes2n$, for some $c$,



$a^2=b^2+2btimes ctimes2n+c^2times2^2n^2$,



$a^2-b^2=(btimes c+c^2n)times4n$, then



$a^2equiv b^2pmod{2^2n}$.



I think that this is right: what I DON’T understand is how to generalize this to:



$aequiv bpmod{kn}Rightarrow a^kequiv b^k pmod{k^2n}$.



Please give me a hint.










share|cite|improve this question















What I have done is this:



$aequiv b pmod{2n}$,



$a=b+ctimes2n$, for some $c$,



$a^2=b^2+2btimes ctimes2n+c^2times2^2n^2$,



$a^2-b^2=(btimes c+c^2n)times4n$, then



$a^2equiv b^2pmod{2^2n}$.



I think that this is right: what I DON’T understand is how to generalize this to:



$aequiv bpmod{kn}Rightarrow a^kequiv b^k pmod{k^2n}$.



Please give me a hint.







number-theory






share|cite|improve this question















share|cite|improve this question













share|cite|improve this question




share|cite|improve this question








edited Jan 4 at 15:33









Bill Dubuque

209k29190631




209k29190631










asked Dec 1 '11 at 10:52









gurghetgurghet

271213




271213




migrated from crypto.stackexchange.com Dec 1 '11 at 13:30


This question came from our site for software developers, mathematicians and others interested in cryptography.






migrated from crypto.stackexchange.com Dec 1 '11 at 13:30


This question came from our site for software developers, mathematicians and others interested in cryptography.














  • Hi @gurghet - I've migrated your question here as it has no direct relation to cryptography as is - so this is the best place to get a good answer.
    – user892
    Dec 1 '11 at 13:32


















  • Hi @gurghet - I've migrated your question here as it has no direct relation to cryptography as is - so this is the best place to get a good answer.
    – user892
    Dec 1 '11 at 13:32
















Hi @gurghet - I've migrated your question here as it has no direct relation to cryptography as is - so this is the best place to get a good answer.
– user892
Dec 1 '11 at 13:32




Hi @gurghet - I've migrated your question here as it has no direct relation to cryptography as is - so this is the best place to get a good answer.
– user892
Dec 1 '11 at 13:32










3 Answers
3






active

oldest

votes


















7














Since $aequiv bpmod{kn}$, we have
$$a=b+ckn$$
for some integer $c$. Now taking the $k$th power on both sides, we have
$$a^k=(b+ckn)^k.$$
By the binomial theorem, the right hand side is given by
$$(b+ckn)^k=b^k+sum_{i=1}^k{kchoose i}(ckn)^ib^{k-i}.$$
For $igeq 2$, it is clear that
${kchoose i}(ckn)^ib^{k-i}$ is divisble by $k^2n$. On the other hand, for $i=1$, we have
${kchoose i}(ckn)^ib^{k-i}=ck^2nb^{k-1}$, which is also divisble by $k^2n$. Therefore, by the above equality, we have
$$(b+ckn)^k=b^k+k^2nN$$
for some integer $N$. Combining all these, we have
$$a^k=(b+ckn)^k=b^k+k^2nN,$$
that is
$$a^kequiv b^k pmod{k^2n}.$$






share|cite|improve this answer



















  • 1




    Thanks but I’m not sure about the binomial theorem. Is that right? I think it misses a $b^{k-i}$ in the sum.
    – gurghet
    Dec 1 '11 at 16:01












  • It works anyway! Thanks!
    – gurghet
    Dec 1 '11 at 16:09










  • @gurghet: Yes, you are right! I missed the term $b^{k-i}$. See my edited answer. Thanks!
    – Paul
    Dec 1 '11 at 21:52



















7














Hint $ $ It's a special case of: $ $ a root of a polynomial is a double root if the derivative vanishes.



Thus to prove that $rm kmid a-b Rightarrow k^2 mid a^k-b^k = (a-b) dfrac{a^k-b^k}{a-b}, $ it suffices to prove



that $rm,k,$ divides the second factor. $ $ Theorem $Rightarrow$ it is $rm,equiv, (a^k)'equiv color{#c00}k, a^{k-1}equiv,0 pmod{!color{#c00}k}. $ QED



Theorem $ $ For $rm R$ a ring and $rm,f(x)in R[x]quad $ [Universal Polynomial Derivative Formula]



$$rmbegin{eqnarray}{}&rm g(x,y) &=&rm frac{f(x)-f(y)}{x-y} in R[x,y]\[.3em]
Rightarrow &rm g(x,x) &=&rm f'(x) in R[x]
end{eqnarray}$$



Proof $ $ By $rm,R$-linearity of the derivative it suffices to verify it for a monomial $rm f(x) = x^k,.$



$$begin{eqnarray}{}rm &rm g(x,y) &=&rm frac{x^k-y^k}{x-y}, = x^{k-1} + x^{k-2},y +, cdots, + x,y^{k-2} + y^{k-1}\[.3em]
Rightarrow &rm g(x,x) &=&rm k x^{k-1} = f'(x)quad {bf QED}
end{eqnarray}qquad $$



Remark $ $ To elaborate on a subtlety raised by a comment, $rm,g(x,y)in R[x,y],$ means that $rm ,g(x,y),$ denotes a polynomial in the indeterminates $rm,x,y,,$ whose coefficients lie in the ring $,R,$ (recall that $rm,x-y,$ divides $rm,f(x)-f(y),$ in $rm,R[x,y],$ by the Factor Theorem). Being a polynomial its value is well-defined for all values of $rm ,x,y,$ so we can "evaluate" it at $rm,y = x,$ (as is exemplified by the concrete case $rm,f(x) = x^k$ used in the proof).



In effect we exploit properties of polynomial rings to universally cancel an "apparent singularity" before evaluating. In more complicated contexts this can lead to nontrivial simplifications, e.g. replacing more complicated topological arguments by trivial algebraic arguments, e.g. see the discussion of the purely algebraic proof of Sylvester's determinant identity linked here.






share|cite|improve this answer



















  • 2




    It is worth emphasizing that $g(x,y)=dfrac{f(x)-f(y)}{x-y}in R[x,y]$ is NOT the imperative "take (f(x)-f(y)) and divide by (x-y)", but the polynomial in two variables (that's what "$in R[x,y]$" means) which when multiplied by (x-y) results in f(x)-f(y) -- this is why g(x,x) is not undefined.
    – Vladimir Sotirov
    Feb 11 '12 at 17:39










  • @VladimirSotirov Actually it does denote the result of that polynomial division. But evaluating it at "apparent singularities" requires that we compute the quotient before we evaluate it. I added a remark elaborating on this subtlety.
    – Bill Dubuque
    Jan 4 at 16:16





















5














Since



$a^k-b^k=(a-b)(a^{k-1}+a^{k-2}b+ldots b^{k-1})$



we need only prove that
$a^{k-1}+a^{k-2}b+ldots + b^{k-1}$ is divisible by $k$. But since $a equiv b (text{mod} k)$ we see that



$ a^{k-1}+a^{k-2}b+ldots + b^{k-1} equiv a^{k-1}+a^{k-1}+ldots +a^{k-1} equiv ka^{k-1} equiv 0 (text{mod} k). $






share|cite|improve this answer





















    Your Answer





    StackExchange.ifUsing("editor", function () {
    return StackExchange.using("mathjaxEditing", function () {
    StackExchange.MarkdownEditor.creationCallbacks.add(function (editor, postfix) {
    StackExchange.mathjaxEditing.prepareWmdForMathJax(editor, postfix, [["$", "$"], ["\\(","\\)"]]);
    });
    });
    }, "mathjax-editing");

    StackExchange.ready(function() {
    var channelOptions = {
    tags: "".split(" "),
    id: "69"
    };
    initTagRenderer("".split(" "), "".split(" "), channelOptions);

    StackExchange.using("externalEditor", function() {
    // Have to fire editor after snippets, if snippets enabled
    if (StackExchange.settings.snippets.snippetsEnabled) {
    StackExchange.using("snippets", function() {
    createEditor();
    });
    }
    else {
    createEditor();
    }
    });

    function createEditor() {
    StackExchange.prepareEditor({
    heartbeatType: 'answer',
    autoActivateHeartbeat: false,
    convertImagesToLinks: true,
    noModals: true,
    showLowRepImageUploadWarning: true,
    reputationToPostImages: 10,
    bindNavPrevention: true,
    postfix: "",
    imageUploader: {
    brandingHtml: "Powered by u003ca class="icon-imgur-white" href="https://imgur.com/"u003eu003c/au003e",
    contentPolicyHtml: "User contributions licensed under u003ca href="https://creativecommons.org/licenses/by-sa/3.0/"u003ecc by-sa 3.0 with attribution requiredu003c/au003e u003ca href="https://stackoverflow.com/legal/content-policy"u003e(content policy)u003c/au003e",
    allowUrls: true
    },
    noCode: true, onDemand: true,
    discardSelector: ".discard-answer"
    ,immediatelyShowMarkdownHelp:true
    });


    }
    });














    draft saved

    draft discarded


















    StackExchange.ready(
    function () {
    StackExchange.openid.initPostLogin('.new-post-login', 'https%3a%2f%2fmath.stackexchange.com%2fquestions%2f87383%2fhow-to-prove-that-if-a-equiv-b-pmodkn-then-ak-equiv-bk-pmodk2n%23new-answer', 'question_page');
    }
    );

    Post as a guest















    Required, but never shown

























    3 Answers
    3






    active

    oldest

    votes








    3 Answers
    3






    active

    oldest

    votes









    active

    oldest

    votes






    active

    oldest

    votes









    7














    Since $aequiv bpmod{kn}$, we have
    $$a=b+ckn$$
    for some integer $c$. Now taking the $k$th power on both sides, we have
    $$a^k=(b+ckn)^k.$$
    By the binomial theorem, the right hand side is given by
    $$(b+ckn)^k=b^k+sum_{i=1}^k{kchoose i}(ckn)^ib^{k-i}.$$
    For $igeq 2$, it is clear that
    ${kchoose i}(ckn)^ib^{k-i}$ is divisble by $k^2n$. On the other hand, for $i=1$, we have
    ${kchoose i}(ckn)^ib^{k-i}=ck^2nb^{k-1}$, which is also divisble by $k^2n$. Therefore, by the above equality, we have
    $$(b+ckn)^k=b^k+k^2nN$$
    for some integer $N$. Combining all these, we have
    $$a^k=(b+ckn)^k=b^k+k^2nN,$$
    that is
    $$a^kequiv b^k pmod{k^2n}.$$






    share|cite|improve this answer



















    • 1




      Thanks but I’m not sure about the binomial theorem. Is that right? I think it misses a $b^{k-i}$ in the sum.
      – gurghet
      Dec 1 '11 at 16:01












    • It works anyway! Thanks!
      – gurghet
      Dec 1 '11 at 16:09










    • @gurghet: Yes, you are right! I missed the term $b^{k-i}$. See my edited answer. Thanks!
      – Paul
      Dec 1 '11 at 21:52
















    7














    Since $aequiv bpmod{kn}$, we have
    $$a=b+ckn$$
    for some integer $c$. Now taking the $k$th power on both sides, we have
    $$a^k=(b+ckn)^k.$$
    By the binomial theorem, the right hand side is given by
    $$(b+ckn)^k=b^k+sum_{i=1}^k{kchoose i}(ckn)^ib^{k-i}.$$
    For $igeq 2$, it is clear that
    ${kchoose i}(ckn)^ib^{k-i}$ is divisble by $k^2n$. On the other hand, for $i=1$, we have
    ${kchoose i}(ckn)^ib^{k-i}=ck^2nb^{k-1}$, which is also divisble by $k^2n$. Therefore, by the above equality, we have
    $$(b+ckn)^k=b^k+k^2nN$$
    for some integer $N$. Combining all these, we have
    $$a^k=(b+ckn)^k=b^k+k^2nN,$$
    that is
    $$a^kequiv b^k pmod{k^2n}.$$






    share|cite|improve this answer



















    • 1




      Thanks but I’m not sure about the binomial theorem. Is that right? I think it misses a $b^{k-i}$ in the sum.
      – gurghet
      Dec 1 '11 at 16:01












    • It works anyway! Thanks!
      – gurghet
      Dec 1 '11 at 16:09










    • @gurghet: Yes, you are right! I missed the term $b^{k-i}$. See my edited answer. Thanks!
      – Paul
      Dec 1 '11 at 21:52














    7












    7








    7






    Since $aequiv bpmod{kn}$, we have
    $$a=b+ckn$$
    for some integer $c$. Now taking the $k$th power on both sides, we have
    $$a^k=(b+ckn)^k.$$
    By the binomial theorem, the right hand side is given by
    $$(b+ckn)^k=b^k+sum_{i=1}^k{kchoose i}(ckn)^ib^{k-i}.$$
    For $igeq 2$, it is clear that
    ${kchoose i}(ckn)^ib^{k-i}$ is divisble by $k^2n$. On the other hand, for $i=1$, we have
    ${kchoose i}(ckn)^ib^{k-i}=ck^2nb^{k-1}$, which is also divisble by $k^2n$. Therefore, by the above equality, we have
    $$(b+ckn)^k=b^k+k^2nN$$
    for some integer $N$. Combining all these, we have
    $$a^k=(b+ckn)^k=b^k+k^2nN,$$
    that is
    $$a^kequiv b^k pmod{k^2n}.$$






    share|cite|improve this answer














    Since $aequiv bpmod{kn}$, we have
    $$a=b+ckn$$
    for some integer $c$. Now taking the $k$th power on both sides, we have
    $$a^k=(b+ckn)^k.$$
    By the binomial theorem, the right hand side is given by
    $$(b+ckn)^k=b^k+sum_{i=1}^k{kchoose i}(ckn)^ib^{k-i}.$$
    For $igeq 2$, it is clear that
    ${kchoose i}(ckn)^ib^{k-i}$ is divisble by $k^2n$. On the other hand, for $i=1$, we have
    ${kchoose i}(ckn)^ib^{k-i}=ck^2nb^{k-1}$, which is also divisble by $k^2n$. Therefore, by the above equality, we have
    $$(b+ckn)^k=b^k+k^2nN$$
    for some integer $N$. Combining all these, we have
    $$a^k=(b+ckn)^k=b^k+k^2nN,$$
    that is
    $$a^kequiv b^k pmod{k^2n}.$$







    share|cite|improve this answer














    share|cite|improve this answer



    share|cite|improve this answer








    edited Dec 1 '11 at 21:51

























    answered Dec 1 '11 at 13:39









    PaulPaul

    15.9k33666




    15.9k33666








    • 1




      Thanks but I’m not sure about the binomial theorem. Is that right? I think it misses a $b^{k-i}$ in the sum.
      – gurghet
      Dec 1 '11 at 16:01












    • It works anyway! Thanks!
      – gurghet
      Dec 1 '11 at 16:09










    • @gurghet: Yes, you are right! I missed the term $b^{k-i}$. See my edited answer. Thanks!
      – Paul
      Dec 1 '11 at 21:52














    • 1




      Thanks but I’m not sure about the binomial theorem. Is that right? I think it misses a $b^{k-i}$ in the sum.
      – gurghet
      Dec 1 '11 at 16:01












    • It works anyway! Thanks!
      – gurghet
      Dec 1 '11 at 16:09










    • @gurghet: Yes, you are right! I missed the term $b^{k-i}$. See my edited answer. Thanks!
      – Paul
      Dec 1 '11 at 21:52








    1




    1




    Thanks but I’m not sure about the binomial theorem. Is that right? I think it misses a $b^{k-i}$ in the sum.
    – gurghet
    Dec 1 '11 at 16:01






    Thanks but I’m not sure about the binomial theorem. Is that right? I think it misses a $b^{k-i}$ in the sum.
    – gurghet
    Dec 1 '11 at 16:01














    It works anyway! Thanks!
    – gurghet
    Dec 1 '11 at 16:09




    It works anyway! Thanks!
    – gurghet
    Dec 1 '11 at 16:09












    @gurghet: Yes, you are right! I missed the term $b^{k-i}$. See my edited answer. Thanks!
    – Paul
    Dec 1 '11 at 21:52




    @gurghet: Yes, you are right! I missed the term $b^{k-i}$. See my edited answer. Thanks!
    – Paul
    Dec 1 '11 at 21:52











    7














    Hint $ $ It's a special case of: $ $ a root of a polynomial is a double root if the derivative vanishes.



    Thus to prove that $rm kmid a-b Rightarrow k^2 mid a^k-b^k = (a-b) dfrac{a^k-b^k}{a-b}, $ it suffices to prove



    that $rm,k,$ divides the second factor. $ $ Theorem $Rightarrow$ it is $rm,equiv, (a^k)'equiv color{#c00}k, a^{k-1}equiv,0 pmod{!color{#c00}k}. $ QED



    Theorem $ $ For $rm R$ a ring and $rm,f(x)in R[x]quad $ [Universal Polynomial Derivative Formula]



    $$rmbegin{eqnarray}{}&rm g(x,y) &=&rm frac{f(x)-f(y)}{x-y} in R[x,y]\[.3em]
    Rightarrow &rm g(x,x) &=&rm f'(x) in R[x]
    end{eqnarray}$$



    Proof $ $ By $rm,R$-linearity of the derivative it suffices to verify it for a monomial $rm f(x) = x^k,.$



    $$begin{eqnarray}{}rm &rm g(x,y) &=&rm frac{x^k-y^k}{x-y}, = x^{k-1} + x^{k-2},y +, cdots, + x,y^{k-2} + y^{k-1}\[.3em]
    Rightarrow &rm g(x,x) &=&rm k x^{k-1} = f'(x)quad {bf QED}
    end{eqnarray}qquad $$



    Remark $ $ To elaborate on a subtlety raised by a comment, $rm,g(x,y)in R[x,y],$ means that $rm ,g(x,y),$ denotes a polynomial in the indeterminates $rm,x,y,,$ whose coefficients lie in the ring $,R,$ (recall that $rm,x-y,$ divides $rm,f(x)-f(y),$ in $rm,R[x,y],$ by the Factor Theorem). Being a polynomial its value is well-defined for all values of $rm ,x,y,$ so we can "evaluate" it at $rm,y = x,$ (as is exemplified by the concrete case $rm,f(x) = x^k$ used in the proof).



    In effect we exploit properties of polynomial rings to universally cancel an "apparent singularity" before evaluating. In more complicated contexts this can lead to nontrivial simplifications, e.g. replacing more complicated topological arguments by trivial algebraic arguments, e.g. see the discussion of the purely algebraic proof of Sylvester's determinant identity linked here.






    share|cite|improve this answer



















    • 2




      It is worth emphasizing that $g(x,y)=dfrac{f(x)-f(y)}{x-y}in R[x,y]$ is NOT the imperative "take (f(x)-f(y)) and divide by (x-y)", but the polynomial in two variables (that's what "$in R[x,y]$" means) which when multiplied by (x-y) results in f(x)-f(y) -- this is why g(x,x) is not undefined.
      – Vladimir Sotirov
      Feb 11 '12 at 17:39










    • @VladimirSotirov Actually it does denote the result of that polynomial division. But evaluating it at "apparent singularities" requires that we compute the quotient before we evaluate it. I added a remark elaborating on this subtlety.
      – Bill Dubuque
      Jan 4 at 16:16


















    7














    Hint $ $ It's a special case of: $ $ a root of a polynomial is a double root if the derivative vanishes.



    Thus to prove that $rm kmid a-b Rightarrow k^2 mid a^k-b^k = (a-b) dfrac{a^k-b^k}{a-b}, $ it suffices to prove



    that $rm,k,$ divides the second factor. $ $ Theorem $Rightarrow$ it is $rm,equiv, (a^k)'equiv color{#c00}k, a^{k-1}equiv,0 pmod{!color{#c00}k}. $ QED



    Theorem $ $ For $rm R$ a ring and $rm,f(x)in R[x]quad $ [Universal Polynomial Derivative Formula]



    $$rmbegin{eqnarray}{}&rm g(x,y) &=&rm frac{f(x)-f(y)}{x-y} in R[x,y]\[.3em]
    Rightarrow &rm g(x,x) &=&rm f'(x) in R[x]
    end{eqnarray}$$



    Proof $ $ By $rm,R$-linearity of the derivative it suffices to verify it for a monomial $rm f(x) = x^k,.$



    $$begin{eqnarray}{}rm &rm g(x,y) &=&rm frac{x^k-y^k}{x-y}, = x^{k-1} + x^{k-2},y +, cdots, + x,y^{k-2} + y^{k-1}\[.3em]
    Rightarrow &rm g(x,x) &=&rm k x^{k-1} = f'(x)quad {bf QED}
    end{eqnarray}qquad $$



    Remark $ $ To elaborate on a subtlety raised by a comment, $rm,g(x,y)in R[x,y],$ means that $rm ,g(x,y),$ denotes a polynomial in the indeterminates $rm,x,y,,$ whose coefficients lie in the ring $,R,$ (recall that $rm,x-y,$ divides $rm,f(x)-f(y),$ in $rm,R[x,y],$ by the Factor Theorem). Being a polynomial its value is well-defined for all values of $rm ,x,y,$ so we can "evaluate" it at $rm,y = x,$ (as is exemplified by the concrete case $rm,f(x) = x^k$ used in the proof).



    In effect we exploit properties of polynomial rings to universally cancel an "apparent singularity" before evaluating. In more complicated contexts this can lead to nontrivial simplifications, e.g. replacing more complicated topological arguments by trivial algebraic arguments, e.g. see the discussion of the purely algebraic proof of Sylvester's determinant identity linked here.






    share|cite|improve this answer



















    • 2




      It is worth emphasizing that $g(x,y)=dfrac{f(x)-f(y)}{x-y}in R[x,y]$ is NOT the imperative "take (f(x)-f(y)) and divide by (x-y)", but the polynomial in two variables (that's what "$in R[x,y]$" means) which when multiplied by (x-y) results in f(x)-f(y) -- this is why g(x,x) is not undefined.
      – Vladimir Sotirov
      Feb 11 '12 at 17:39










    • @VladimirSotirov Actually it does denote the result of that polynomial division. But evaluating it at "apparent singularities" requires that we compute the quotient before we evaluate it. I added a remark elaborating on this subtlety.
      – Bill Dubuque
      Jan 4 at 16:16
















    7












    7








    7






    Hint $ $ It's a special case of: $ $ a root of a polynomial is a double root if the derivative vanishes.



    Thus to prove that $rm kmid a-b Rightarrow k^2 mid a^k-b^k = (a-b) dfrac{a^k-b^k}{a-b}, $ it suffices to prove



    that $rm,k,$ divides the second factor. $ $ Theorem $Rightarrow$ it is $rm,equiv, (a^k)'equiv color{#c00}k, a^{k-1}equiv,0 pmod{!color{#c00}k}. $ QED



    Theorem $ $ For $rm R$ a ring and $rm,f(x)in R[x]quad $ [Universal Polynomial Derivative Formula]



    $$rmbegin{eqnarray}{}&rm g(x,y) &=&rm frac{f(x)-f(y)}{x-y} in R[x,y]\[.3em]
    Rightarrow &rm g(x,x) &=&rm f'(x) in R[x]
    end{eqnarray}$$



    Proof $ $ By $rm,R$-linearity of the derivative it suffices to verify it for a monomial $rm f(x) = x^k,.$



    $$begin{eqnarray}{}rm &rm g(x,y) &=&rm frac{x^k-y^k}{x-y}, = x^{k-1} + x^{k-2},y +, cdots, + x,y^{k-2} + y^{k-1}\[.3em]
    Rightarrow &rm g(x,x) &=&rm k x^{k-1} = f'(x)quad {bf QED}
    end{eqnarray}qquad $$



    Remark $ $ To elaborate on a subtlety raised by a comment, $rm,g(x,y)in R[x,y],$ means that $rm ,g(x,y),$ denotes a polynomial in the indeterminates $rm,x,y,,$ whose coefficients lie in the ring $,R,$ (recall that $rm,x-y,$ divides $rm,f(x)-f(y),$ in $rm,R[x,y],$ by the Factor Theorem). Being a polynomial its value is well-defined for all values of $rm ,x,y,$ so we can "evaluate" it at $rm,y = x,$ (as is exemplified by the concrete case $rm,f(x) = x^k$ used in the proof).



    In effect we exploit properties of polynomial rings to universally cancel an "apparent singularity" before evaluating. In more complicated contexts this can lead to nontrivial simplifications, e.g. replacing more complicated topological arguments by trivial algebraic arguments, e.g. see the discussion of the purely algebraic proof of Sylvester's determinant identity linked here.






    share|cite|improve this answer














    Hint $ $ It's a special case of: $ $ a root of a polynomial is a double root if the derivative vanishes.



    Thus to prove that $rm kmid a-b Rightarrow k^2 mid a^k-b^k = (a-b) dfrac{a^k-b^k}{a-b}, $ it suffices to prove



    that $rm,k,$ divides the second factor. $ $ Theorem $Rightarrow$ it is $rm,equiv, (a^k)'equiv color{#c00}k, a^{k-1}equiv,0 pmod{!color{#c00}k}. $ QED



    Theorem $ $ For $rm R$ a ring and $rm,f(x)in R[x]quad $ [Universal Polynomial Derivative Formula]



    $$rmbegin{eqnarray}{}&rm g(x,y) &=&rm frac{f(x)-f(y)}{x-y} in R[x,y]\[.3em]
    Rightarrow &rm g(x,x) &=&rm f'(x) in R[x]
    end{eqnarray}$$



    Proof $ $ By $rm,R$-linearity of the derivative it suffices to verify it for a monomial $rm f(x) = x^k,.$



    $$begin{eqnarray}{}rm &rm g(x,y) &=&rm frac{x^k-y^k}{x-y}, = x^{k-1} + x^{k-2},y +, cdots, + x,y^{k-2} + y^{k-1}\[.3em]
    Rightarrow &rm g(x,x) &=&rm k x^{k-1} = f'(x)quad {bf QED}
    end{eqnarray}qquad $$



    Remark $ $ To elaborate on a subtlety raised by a comment, $rm,g(x,y)in R[x,y],$ means that $rm ,g(x,y),$ denotes a polynomial in the indeterminates $rm,x,y,,$ whose coefficients lie in the ring $,R,$ (recall that $rm,x-y,$ divides $rm,f(x)-f(y),$ in $rm,R[x,y],$ by the Factor Theorem). Being a polynomial its value is well-defined for all values of $rm ,x,y,$ so we can "evaluate" it at $rm,y = x,$ (as is exemplified by the concrete case $rm,f(x) = x^k$ used in the proof).



    In effect we exploit properties of polynomial rings to universally cancel an "apparent singularity" before evaluating. In more complicated contexts this can lead to nontrivial simplifications, e.g. replacing more complicated topological arguments by trivial algebraic arguments, e.g. see the discussion of the purely algebraic proof of Sylvester's determinant identity linked here.







    share|cite|improve this answer














    share|cite|improve this answer



    share|cite|improve this answer








    edited Jan 4 at 17:20

























    answered Jan 23 '12 at 6:01









    Bill DubuqueBill Dubuque

    209k29190631




    209k29190631








    • 2




      It is worth emphasizing that $g(x,y)=dfrac{f(x)-f(y)}{x-y}in R[x,y]$ is NOT the imperative "take (f(x)-f(y)) and divide by (x-y)", but the polynomial in two variables (that's what "$in R[x,y]$" means) which when multiplied by (x-y) results in f(x)-f(y) -- this is why g(x,x) is not undefined.
      – Vladimir Sotirov
      Feb 11 '12 at 17:39










    • @VladimirSotirov Actually it does denote the result of that polynomial division. But evaluating it at "apparent singularities" requires that we compute the quotient before we evaluate it. I added a remark elaborating on this subtlety.
      – Bill Dubuque
      Jan 4 at 16:16
















    • 2




      It is worth emphasizing that $g(x,y)=dfrac{f(x)-f(y)}{x-y}in R[x,y]$ is NOT the imperative "take (f(x)-f(y)) and divide by (x-y)", but the polynomial in two variables (that's what "$in R[x,y]$" means) which when multiplied by (x-y) results in f(x)-f(y) -- this is why g(x,x) is not undefined.
      – Vladimir Sotirov
      Feb 11 '12 at 17:39










    • @VladimirSotirov Actually it does denote the result of that polynomial division. But evaluating it at "apparent singularities" requires that we compute the quotient before we evaluate it. I added a remark elaborating on this subtlety.
      – Bill Dubuque
      Jan 4 at 16:16










    2




    2




    It is worth emphasizing that $g(x,y)=dfrac{f(x)-f(y)}{x-y}in R[x,y]$ is NOT the imperative "take (f(x)-f(y)) and divide by (x-y)", but the polynomial in two variables (that's what "$in R[x,y]$" means) which when multiplied by (x-y) results in f(x)-f(y) -- this is why g(x,x) is not undefined.
    – Vladimir Sotirov
    Feb 11 '12 at 17:39




    It is worth emphasizing that $g(x,y)=dfrac{f(x)-f(y)}{x-y}in R[x,y]$ is NOT the imperative "take (f(x)-f(y)) and divide by (x-y)", but the polynomial in two variables (that's what "$in R[x,y]$" means) which when multiplied by (x-y) results in f(x)-f(y) -- this is why g(x,x) is not undefined.
    – Vladimir Sotirov
    Feb 11 '12 at 17:39












    @VladimirSotirov Actually it does denote the result of that polynomial division. But evaluating it at "apparent singularities" requires that we compute the quotient before we evaluate it. I added a remark elaborating on this subtlety.
    – Bill Dubuque
    Jan 4 at 16:16






    @VladimirSotirov Actually it does denote the result of that polynomial division. But evaluating it at "apparent singularities" requires that we compute the quotient before we evaluate it. I added a remark elaborating on this subtlety.
    – Bill Dubuque
    Jan 4 at 16:16













    5














    Since



    $a^k-b^k=(a-b)(a^{k-1}+a^{k-2}b+ldots b^{k-1})$



    we need only prove that
    $a^{k-1}+a^{k-2}b+ldots + b^{k-1}$ is divisible by $k$. But since $a equiv b (text{mod} k)$ we see that



    $ a^{k-1}+a^{k-2}b+ldots + b^{k-1} equiv a^{k-1}+a^{k-1}+ldots +a^{k-1} equiv ka^{k-1} equiv 0 (text{mod} k). $






    share|cite|improve this answer


























      5














      Since



      $a^k-b^k=(a-b)(a^{k-1}+a^{k-2}b+ldots b^{k-1})$



      we need only prove that
      $a^{k-1}+a^{k-2}b+ldots + b^{k-1}$ is divisible by $k$. But since $a equiv b (text{mod} k)$ we see that



      $ a^{k-1}+a^{k-2}b+ldots + b^{k-1} equiv a^{k-1}+a^{k-1}+ldots +a^{k-1} equiv ka^{k-1} equiv 0 (text{mod} k). $






      share|cite|improve this answer
























        5












        5








        5






        Since



        $a^k-b^k=(a-b)(a^{k-1}+a^{k-2}b+ldots b^{k-1})$



        we need only prove that
        $a^{k-1}+a^{k-2}b+ldots + b^{k-1}$ is divisible by $k$. But since $a equiv b (text{mod} k)$ we see that



        $ a^{k-1}+a^{k-2}b+ldots + b^{k-1} equiv a^{k-1}+a^{k-1}+ldots +a^{k-1} equiv ka^{k-1} equiv 0 (text{mod} k). $






        share|cite|improve this answer












        Since



        $a^k-b^k=(a-b)(a^{k-1}+a^{k-2}b+ldots b^{k-1})$



        we need only prove that
        $a^{k-1}+a^{k-2}b+ldots + b^{k-1}$ is divisible by $k$. But since $a equiv b (text{mod} k)$ we see that



        $ a^{k-1}+a^{k-2}b+ldots + b^{k-1} equiv a^{k-1}+a^{k-1}+ldots +a^{k-1} equiv ka^{k-1} equiv 0 (text{mod} k). $







        share|cite|improve this answer












        share|cite|improve this answer



        share|cite|improve this answer










        answered Dec 1 '11 at 14:54









        JohanJohan

        1,2011821




        1,2011821






























            draft saved

            draft discarded




















































            Thanks for contributing an answer to Mathematics Stack Exchange!


            • Please be sure to answer the question. Provide details and share your research!

            But avoid



            • Asking for help, clarification, or responding to other answers.

            • Making statements based on opinion; back them up with references or personal experience.


            Use MathJax to format equations. MathJax reference.


            To learn more, see our tips on writing great answers.





            Some of your past answers have not been well-received, and you're in danger of being blocked from answering.


            Please pay close attention to the following guidance:


            • Please be sure to answer the question. Provide details and share your research!

            But avoid



            • Asking for help, clarification, or responding to other answers.

            • Making statements based on opinion; back them up with references or personal experience.


            To learn more, see our tips on writing great answers.




            draft saved


            draft discarded














            StackExchange.ready(
            function () {
            StackExchange.openid.initPostLogin('.new-post-login', 'https%3a%2f%2fmath.stackexchange.com%2fquestions%2f87383%2fhow-to-prove-that-if-a-equiv-b-pmodkn-then-ak-equiv-bk-pmodk2n%23new-answer', 'question_page');
            }
            );

            Post as a guest















            Required, but never shown





















































            Required, but never shown














            Required, but never shown












            Required, but never shown







            Required, but never shown

































            Required, but never shown














            Required, but never shown












            Required, but never shown







            Required, but never shown







            Popular posts from this blog

            An IMO inspired problem

            Management

            Investment